If T delivers the third speech, which one of the following must be true?

ari on September 28, 2020

Can someone explain to me how question two is J in the fifth spot.

I thought that since it said HJK and RST cannot go in a consecutive order we would have to split each of them with the other letters. If T goes third, wouldn't that mean an R or an S go in the fifth spot. I think I am just confused on the two rules.

Reply
Create a free account to read and take part in forum discussions.

Already have an account? log in

shunhe on October 2, 2020

Hi @ari,

Thanks for the question! So we’re asked to make T give the third speech and then asked what has to be true based on that. Well, we know that J can’t be before or after T based on the last rule. So that means J can’t be in slots 2 or 4. Take a look at the fourth rule. J also can’t be 1st or 6th. So J can’t be 1, 2, 4, or 6, and it can’t be 3 either, since T is already there. J has to be somewhere, so it’s in 5. And that’s why (B) is the correct answer.

Hope this helps! Feel free to ask any other questions that you might have.